Pagina 1 di 1

Da un vecchio IMO 1975, e poi anche 1978, disuguaglianze

Inviato: 06 giu 2007, 12:25
da salva90
Siano $ ~x_i, ~y_i $ numeri reali tali che $ x_1\ge x_2\ge\cdots\ge x_n $ e $ y_1\ge y_2\ge\cdots\ge y_n $.
Siano $ ~z_i $ permutazioni degli $ ~y_i $; provare che:

$ \displaystyle \sum_{i=1}^n (x_i-y_i)^2\le\sum_{i=1}^n(x_i-z_i)^2 $

Era il numero 1 delle IMO 1975... e non è difficile... dedicato a chi, come me, sta muoendo i primi passi nelle disuguaglianze :wink:

Inviato: 06 giu 2007, 17:10
da marco-daddy
Direi che è un semplice riarrangiamento

:wink:

Inviato: 06 giu 2007, 18:25
da Sherlock
è giusto, muahahahahahahah

Inviato: 06 giu 2007, 18:33
da EUCLA
e io che credevo che valesse solo per i prodotti...stavo giusto cercando una dimostrazione per le somme/differenze....povera me

Inviato: 06 giu 2007, 19:54
da dovix91
Infatti in questo caso lo usi per 1 sommatoria di prodotti...
svolgendo i quadrati e poi semplificando ti ritrovi a riarrangiamento... :wink:
[/quote]

Inviato: 06 giu 2007, 19:58
da edriv
Eddai ma chi vi ha insegnato a scrivere tutto in bianco? Uffa!

Scusate l'antipatica intrusione :?

Inviato: 06 giu 2007, 20:12
da EUCLA
rimane comunque il povera me
:D

Inviato: 06 giu 2007, 20:13
da Sherlock
vai via intruso!!

Inviato: 06 giu 2007, 20:18
da salva90
Ok, visto che questo topic l'avete distrutto rilancio con un IMO 1978:

Siano $ a_1, a_2, \dots, a_n $ interi positivi tutti distinti tra loro; provare che:

$ \displaystyle\sum_{k=1}^n\frac{a_k}{k^2}\ge\sum_{k=1}^n\frac1k $

due appunti:
1- astenersi esperti; astenersi anche Marco Daddy visto il modo in cui ha ammazzato l'altra
2- qualcuno NON ESPERTO, dopo averla risolta, scriva la soluzione per bene senza queste cavolo di citazioni...

Inviato: 06 giu 2007, 20:47
da Poliwhirl
Sono grande, ma non sono esperto, ed è tanto che non posto una soluzione,
dunque... (spero di non sbagliare come mio solito) :)

Parte I: sia $ \displaystyle \sigma: \{a_1,\dots,a_n\} \rightarrow \{b_1,\dots,b_n\} $ una permutazione degli $ a_i $ tale che $ b_1 < b_2 < \dots <b_n $, per riarrangiamento si ha $ \displaystyle \sum_{k=1}^{n} \frac{a_k}{k^2} \geq \sum_{k=1}^{n} \frac{b_k}{k^2} $.

Parte II: se $ b_1 < b_2 < \dots < b_n $ allora $ \displaystyle \forall k \in \{1,\dots,n\}, b_k \geq k $; per $ k=1 $ deve essere $ b_1 \geq 1 $, vera perchè il più piccolo valore che può assumere un $ b_i $ è 1 per ipotesi; sia vero il lemma per k e dimostriamolo per k+1: $ \displaystyle b_k \geq k \Rightarrow b_k + 1 \geq k + 1 \Rightarrow b_{k+1}\geq b_k+1 \geq k+1 \Rightarrow b_{k+1} \geq k+1 $; per induzione la parte II è dimostrata.

Parte III: Da II deduciamo che $ \displaystyle \forall k \in \{1,...,n\}, \frac{b_k}{k} \geq 1 $ da cui $ \displaystyle \forall k \in \{1,...,n\}, \frac{b_k}{k^2} \geq \frac{1}{k} \Rightarrow \sum_{k=1}^{n} \frac{b_k}{k^2} \geq \sum_{k=1}^{n} \frac{1}{k} $.

Epilogo: da parte I e parte III segue $ \displaystyle \sum_{k=1}^{n} \frac{a_k}{k^2} \geq \sum_{k=1}^{n} \frac{b_k}{k^2} \geq \sum_{k=1}^{n} \frac{1}{k} $ c.v.d..

Bye,
#Poliwhirl#

[EDIT: corretto, forse]

Inviato: 06 giu 2007, 20:51
da Sisifo
Poliwhirl ha scritto:Lemma: WLOG $ a_1 < a_2 < \dots < a_n $
Non voglio smontarti ma.. questo WLOG non ci sta proprio.. anzi è il punto centrale dell'esercizio giustificarlo in maniera decente, non dico come se no Salva mi spara.. Il fatto è che il membro di sinistra non è simmetrico nelle $ a_1, \dots, a_n $ quindi non puoi riordinarle a piacere..

Inviato: 07 giu 2007, 22:04
da salva90
La mia soluzione è piuttosto simile a quella di Poliwhirl, ma non fa uso dell'induzione. Essendo questi i miei primi passi nelle disuguaglianze la posto così qualcuno mi dirà se va bene...

Per prima cosa riscrivo il RHS come $ \sum_k k \cdot \frac 1{k^2} $
Ora considero una permutazione $ \sigma(k) $ tale che $ a_{\sigma(1)}<a_{\sigma(2)}<\cdots<a_{\sigma(n)} $ e chiamo $ \tau(k) $ la sua inversa. A questo punto è chiaro che, data una n-upla arbitraria di reali positivi $ (x_1, x_2, \dots, x_n) $, si ha $ \displaystyle\sum_{k=1}^n a_kx_k\ge\sum_{k=1}^n\tau(k)x_k $, poichè $ a_{\sigma(k)}\ge k~\forall k $

Perciò abbiamo:
$ \displaystyle\sum_{i=1}^n a_k\cdot \frac 1 {k^2}\ge\sum_{i=1}^n \tau(k)\cdot\frac 1{k^2}\ge\sum_{i=1}^n k\cdot\frac1{k^2} $
dove l'ultima disuguaglianza è giustificata dal riarrangiamento sulle n-uple $ (1, 2, \dots, n) $; $ (\frac11, \frac 14, \dots, \frac1{n^2}) $ del quale il RHS rappresenta il caso minimo

Inviato: 07 ago 2007, 23:53
da Sepp
Vediamo quanti errori riesco a fare in 3 righe.. :D

Poniamo $ S = \sum_{k} \frac{\frac{1}{k^2}}{\frac{1}{a_{k}}} $.

Per Cauchy $ S \geq \frac{\left(\sum_{k} \frac{1}{k}\right)^2}{\sum_{k} \frac{1}{a_{k}}} $.

Ma abbiamo $ \sum_{k} \frac{1}{k} \geq \sum_{k} \frac{1}{a_{k}} $, da cui la tesi.